w23-logic-2/inputs/lecture_16.tex

465 lines
16 KiB
TeX
Raw Normal View History

2023-12-19 03:43:49 +01:00
<<<<<<< HEAD
2023-12-11 15:45:36 +01:00
\lecture{16}{2023-12-11}{}
Recall \yaref{thm:fodor}.
\begin{question}
What happens if $S$ is nonstationary?
\end{question}
Let $S \subseteq \kappa$ be nonstationary,
$\kappa$ uncounable and regular.
Then there is a club $C \subseteq \kappa$
with $C \cap S = \emptyset$.
Let us define $f\colon S \to \kappa$ in the following way:
If $\alpha \in S$ and $C \cap \alpha \neq \emptyset$,
then $\max(C \cap \alpha) < \alpha$.
Define
\[
f(\alpha) \coloneqq \begin{cases}
0 &: C \cap \alpha = \emptyset,\\
\max(C \cap \alpha) &:C\cap \alpha \neq \emptyset.
\end{cases}
\]
For all $\alpha > 0$,
we have that $f(\alpha) < \alpha$.
If $\gamma \in \ran(f)$ then
$f(\alpha) = \gamma$ implies either $\gamma = 0$ and $\alpha < \min(C)$
or $\gamma \in C$ and $\gamma < \alpha < \gamma'$
where $\gamma' = \min(C \setminus (\gamma + 1))$.
Thus for all $\gamma$,
there is only an interval of ordinals $\alpha \in S$
where $f(\alpha) = \gamma$.
\todo{Move this to the definition of filter}
Recall that $F \subseteq \cP(\kappa)$ is a filter if
$X,Y \in F \implies X \cap Y \in F$,
$X \in X, X \subseteq Y \subseteq \kappa \implies Y \in F$
and $\emptyset \not\in F, \kappa \in F$.
\begin{definition}
A filter $F$ is an \vocab{ultrafilter}
iff for all $X \subseteq \kappa$
either $X \in F$ or $\kappa \setminus X \in F$.
\end{definition}
\begin{example}
Examples of filters:
\begin{enumerate}[(a)]
\item Let $\kappa \ge \aleph_0$
and let $F = \{X \subseteq \kappa: \kappa \setminus X \text{ is finite}\}$.
This is called the \vocab{Fr\'echet filter}
or \vocab{cofinal filter}.
It is not an ultrafilter
(consider for example the even and odd numbers\footnote{we consider limit ordinals to be even}).
\item Let $\kappa$ be uncountable and regular.
Then $\cF_\kappa \coloneqq \{X \subseteq \kappa: \exists C \subseteq \kappa \text{ club in $\kappa$}. C \subseteq X\}$.
\end{enumerate}
\end{example}
\begin{question}
Is $\cF_\kappa$ an ultrafilter?
\end{question}
This is certainly not the case if $\kappa \ge \aleph_2$,
because then $S_0 \coloneqq \{\alpha < \kappa : \cf(\alpha) = \omega\}$
and $S_1 \coloneqq \{\alpha < \kappa : \cf(\alpha) = \omega_1\} $
are both stationary
and clearly disjoint.
So neither $S_0$ nor $S_1 \subseteq \kappa \setminus S_0$ contains a club.
For $\kappa < \aleph_1$
this argument does not work, since there is only
on cofinality.
\begin{theorem}[Solovay]
\yalabel{Solovay's Theorem}{Solovay}{thm:solovay}
Let $\kappa$ be regular and uncountable.
If $S \subseteq \kappa$
is stationary,
there is a sequence $\langle S_i : i < \kappa \rangle$
of pairwise disjoint stationary sets of $\kappa$
such that $S = \bigcup S_i$.
\end{theorem}
\begin{corollary}
$\cF_{\aleph_1}$ is not an ultrafilter.
\end{corollary}
\begin{proof}
Apply \yaref{thm:solovay} to $S = \aleph_1$.
Let $\aleph_1 = A \cup B$
where $A$ and $B$ are both stationary
and disjoint.
Then use the argument from above.
\end{proof}
\begin{refproof}{thm:solovay}%
%\footnote{``This is one of the arguments where it is certainly
% worth it to look at it again''}
% TODO: Look at this again and think about it.
We will only proof this for $\aleph_1$.
Fix $S \subseteq \aleph_1$ stationary.
For each $0 < \alpha < \omega_1$,
either $\alpha$ is a successor ordinal
or $\alpha$ is a limit ordinal and $\cf(\alpha) = \omega_1$.
Let $S^\ast \coloneqq \{\alpha \in S \setminus \{0\} : \alpha \text{ is a limit ordinal}\} $.
$S^\ast$ is still stationary:
Let $C \subseteq \omega_1$
be a club,
then $D = \{\alpha \in C \setminus \{0\} : \alpha \text{ is a limit ordinal}\} $
is still a club,
so
\[S^\ast \cap C = S^\ast \cap D = S \cap D \neq \emptyset.\]
Let
\[
\langle \langle \gamma_n^{\alpha} : n < \omega \rangle : \alpha \in S^\ast\rangle
\]
be such that $ \langle \gamma_n^\alpha : n < \omega \rangle$
is cofinal in $ \alpha$.
\begin{claim}
\label{thm:solovay:p:c1}
There exists $n < \omega$
such that for all $\delta < \omega_1$
the set
\[
\{\alpha \in S^\ast : \gamma_n^\alpha > \delta\}
\]
is stationary.
\end{claim}
\begin{subproof}
Otherwise for all $n < \omega$,
there is a $\delta$ such that
$\{\alpha \in S^\ast : \gamma_n^{\alpha} > \delta\}$
is nonstationary.
Let $\delta_n$ be the least such $\delta$.
Let $C_n$ be a club disjoint from
\[
\{\alpha \in S^\ast : \gamma_n^{\alpha} > \delta_n\},
\]
i.e.~if $\alpha \in S^\ast \cap C_n$, then $\gamma_n^{\alpha} \le \delta_n$.
Let $\delta^\ast \coloneqq \sup_{n< \omega}\delta_n$.
Let $C = \bigcap_{n < \omega} C_n$.
Then $C$ is a club.
We must have that if $\alpha \in S^\ast \cap C$
then $\gamma_n^{\alpha} \le \delta^\ast$ for all $n$.
% But now things get a bit fishy:
Let $C' \coloneqq C \setminus (\delta^\ast + 1)$.
$C'$ is still club.
As $\delta^\ast$ is stationary,
we may pick some $\alpha \in S^\ast \cap C'$.
But then $\gamma_n^{\alpha} > \delta^\ast$
for $n$ large enough
as $\langle \gamma_n^{\alpha} : n < \omega \rangle$
is cofinal in $\alpha$ $\lightning$.
\end{subproof}
Let $n < \omega$ be as in \yaref{thm:solovay:p:c1}.
Consider
\begin{IEEEeqnarray*}{rCl}
f\colon S^\ast&\longrightarrow & \omega_1\\
\alpha&\longmapsto & \gamma^{\alpha}_n.
\end{IEEEeqnarray*}
Clearly this is regressive.
We will now define a strictly increasing sequence
$\langle \delta_i : i < \omega_1 \rangle$
as follows:
Let $\delta_0 = 0$.
For $0 < i < \omega_1$ suppose that $\delta_j, j < i$ have been defined.
Let $\delta \coloneqq (\sup_{j < i} \delta_j) + 1$.
By \yaref{thm:solovay:p:c1} (rather, by the choice of $n$),
we have that $\{\alpha \in S^\ast : \gamma_n^{\alpha} > \delta\}$
is stationary.
Hence by Fodor there is some stationary $T \subseteq S^\ast$
and some $\delta'$ such that for all $\alpha \in T$
we have
$\gamma_n^{\alpha} = \delta'$.
Write $\delta_i = \delta'$ and $T_i = T$.
\begin{claim}
\label{thm:solovay:p:c2}
Each $T_i$ is stationary
and if $i \neq j$, then $T_i \cap T_j = \emptyset$.
\end{claim}
\begin{subproof}
The first part is true by construction.
Let $j < i$.
Then if $\alpha \in T_i$, $\alpha' \in T_j$,
we get $\gamma_n^{\alpha'} = \delta_j < \delta_i = \gamma_n^{\alpha}$
hence $\alpha \neq \alpha'$.
\end{subproof}
Now let
\[
S_i \coloneqq \begin{cases}
T_i &: i > 0,\\
T_0 \cup (S \setminus \bigcup_{j > 0} T_j) &: i = 0.
\end{cases}
\]
Then $\langle S_i : i < \omega_1 \rangle$ is
as desired.
\end{refproof}
We now want to do another application of \yaref{thm:fodor}.
Recall that $2^{\kappa} > \kappa$, in fact $\cf(2^{\kappa}) > \kappa$
by \yaref{thm:koenig}.
Trivially, if $\kappa \le \lambda$ then $2^{\kappa} \le 2^{\lambda}$.
This is in some sense the only thing we can prove about successor cardinals.
However we can say something about singular cardinals:
\begin{theorem}[Silver]
\yaref{Silver's Theorem}{Silver}{thm:silver}
Let $\kappa$ be a singular cardinal of uncountable cofinality.
Assume that $2^{\lambda} = \lambda^+$ for all (infinite)
cardinals $\lambda < \kappa$.
Then $2^{\kappa} = \kappa^+$.
\end{theorem}
\begin{definition}
$\GCH$, the \vocab{generalized continuum hypothesis}
is the statement
that $2^{\lambda} = \lambda^+$ holds for all infinite cardinals $\lambda$,
\end{definition}
Recall that $\CH$ says that $2^{\aleph_0} = \aleph_1$.
So $\GCH \implies \CH$.
\yalabel{thm:silver} says that if $\GCH$ is true below $\kappa$,
then it is true at $\kappa$.
The proof of \yalabel{thm:silver} is quite elementary,
so we will do it now, but the statement can only be fully appreciated later.
2023-12-19 03:43:49 +01:00
||||||| ede97ee
=======
2023-12-15 05:26:57 +01:00
\lecture{16}{2023-12-14}{Silver's Theorem}
We now want to prove \yaref{thm:silver}.
More generally, if $\kappa$ is a singular cardinal of uncountable cofinality
such that $2^{\lambda} = \lambda^+$ for all $\lambda < \kappa$,
then $2^{\kappa} = \kappa^+$.
\begin{remark}
The hypothesis of \yaref{thm:silver}
is consistent with $\ZFC$.
\end{remark}
We will only proof \yaref{thm:silver} in the special case that $\kappa = \aleph_{\omega_1}$.
The general proof differs only in notation.
\begin{remark}
It is important that the cofinality is uncountable.
For example it is consistent
with $\ZFC$ that
$2^{\aleph_n} = \aleph_{n+1}$ for all $n < \omega$
but at the same time $2^{\aleph_{\omega}} = \aleph_{\omega + 2}$.
\end{remark}
\begin{refproof}{thm:silver}
We need to count the number of $X \subseteq \aleph_{\omega_1}.$
Let us fix $\langle f_\lambda : \lambda < \kappa \text{ an infinite cardinal} \rangle$
such that $f_{\lambda}\colon \cP(\lambda) \to \lambda^+$
is bijective for each $\lambda < \kappa$.
For $X \subseteq \aleph_{\omega_1}$
define
\begin{IEEEeqnarray*}{rCl}
f_X\colon \omega_1 &\longrightarrow & \aleph_{\omega_1} \\
\alpha &\longmapsto & f_{\aleph_\alpha}(X \cap \aleph_\alpha).
\end{IEEEeqnarray*}
\begin{claim}
For $X,Y \subseteq \aleph_{\omega_1}$
it is $X \neq Y \iff f_X \neq f_Y$.
\end{claim}
\begin{subproof}
$X \neq Y$ holds iff $X \cap \aleph_\alpha \neq Y \cap \aleph_\alpha$
for some $\alpha < \omega_1$.
But then $f_X(\alpha) \neq f_Y(\alpha)$.
\end{subproof}
For $X, Y \subseteq \aleph_{\omega_1}$
write $X \le Y$ iff
\[
\{\alpha < \omega_1 : f_X(\alpha) \le f_Y(\alpha)\}
\]
is stationary.
\begin{claim}
For all $X,Y \subseteq \aleph_{\omega_1}$,
$X \le Y$ or $Y \le X$.
\end{claim}
\begin{subproof}
Suppose that $X \nleq Y$ and $Y \nleq X$.
Then there are clubs $C,D \subseteq \omega_1$
such that
\[
C \cap \{\alpha < \omega_1 : f_X(\alpha) \le f_Y(\alpha)\} = \emptyset
\]
and
\[
D \cap \{\alpha < \omega_1 : f_Y(\alpha) \le f_X(\alpha)\} = \emptyset.
\]
Note that $C \cap D$ is a club.
Take some $\alpha \in C \cap D$.
But then $f_X(\alpha) \le f_Y(\alpha)$
or $f_{Y}(\alpha) \le f_X(\alpha)$ $\lightning$
\end{subproof}
\begin{claim}
\label{thm:silver:p:c3}.
Let $X \subseteq \aleph_{\omega_1}$.
Then
\[
|\{Y \subseteq \aleph_{\omega_1} : Y \le X\}| \le \aleph_{\omega_1}.
\]
\end{claim}
\begin{subproof}
Write $A \coloneqq \{Y \subseteq X_{\omega_1} : Y \le X\}$.
Suppose $|A| \ge \aleph_{\omega_1 + 1}$.
For each $Y \in A$
we have that
\[
S_Y \coloneqq \{\alpha : f_Y(\alpha) \le f_X(\alpha)\}
\]
is a stationary subset of $\omega_1$.
Since by assumption $2^{\aleph_1} = \aleph_2$,
there are at most $\aleph_2$ such $S_Y$.
Suppose that for each $S \subseteq \omega_1$,
\[
|\{Y \in A : S_Y = S\}| < \aleph_{\omega_1 + 1}.
\]
Then $A$ is the union of $\le \aleph_2$ many
sets of size $< \aleph_{\omega_1 + 1}$.
Thus this is a contradiction since $\aleph_{\omega_1 + 1}$
is regular.
So there exists a stationary $S \subseteq \omega_1$
such that
\[
A_1 = \{Y \subseteq \aleph_{\omega_1} : S_Y = S\}
\]
has cardinality $\aleph_{\omega_1 + 1}$.
We have
\[f_Y(\alpha) \le f_X(\alpha) = f_{\aleph_\alpha}(X \cap \aleph_\alpha)< \aleph_{\alpha + 1}\]
for all $Y \in A_1, \alpha \in S$.
Let $\langle g_{\alpha} : \alpha \in S \rangle$
be such that $g_\alpha\colon \aleph_{\alpha} \twoheadrightarrow f_X(\alpha) + 1$
is a surjection for all $\alpha \in S$.
Then for each $Y \in A_1$ define
\begin{IEEEeqnarray*}{rCl}
\overline{f}_Y\colon S &\longrightarrow & \aleph_{\omega_1} \\
\alpha &\longmapsto & \min \{\xi : g_\alpha(\xi) = f_Y(\alpha)\}.
\end{IEEEeqnarray*}
Let $D$ be the set of all limit ordinals $< \omega_1$.
Then $S \cap D$ is a stationary set:
If $C$ is a club, then $C \cap D$ is a club,
hence $(S \cap D) \cap C = S \cap (D \cap C) \neq \emptyset$.
Now to each $Y \in A$ we may associate
a regressive function
\begin{IEEEeqnarray*}{rCl}
h_Y \colon S \cap D &\longrightarrow & \omega_1 \\
\alpha &\longmapsto & \min \{\beta < \alpha : \overline{f}_Y(\alpha) < \aleph_{\beta}\}.
\end{IEEEeqnarray*}
$h_Y$ is regressive, so by \yaref{thm:fodor}
there is a stationary $T_Y \subseteq S \cap D$ on which $h_Y$ is constant.
By an argument as before,
there is a stationary $T \subseteq S \cap D$ such that
\[
|A_2| = \aleph_{\omega_1 +1},
\]
where $A_2 \coloneqq \{Y \in A_1 : T_Y = T\}$.
Let $\beta < \omega_1$ be such that for all $Y \in A_2$
and for all $\alpha \in T$, $h_Y(\alpha) = \beta$.
Then $\overline{f}_Y(\alpha) < \aleph_\beta$
for all $Y \in A_2$ and $\alpha \in T$.
There are at most $\aleph_\beta^{\aleph_1}$ many functions
$T \to \aleph_\beta$,
but
\begin{IEEEeqnarray*}{rCl}
\aleph_\beta^{\aleph_1} &\le & 2^{\aleph_\beta \cdot \aleph_1}\\
&=& \aleph_{\beta+1} \cdot \aleph_2\\
&<& \aleph_{\omega_1}.
\end{IEEEeqnarray*}
Suppose that for each function
$\tilde{f}\colon T \to \aleph_\beta$
there are $< \aleph_{\omega_1 + 1}$ many $Y \in A_2$
with $\overline{f}_Y \cap T = \tilde{f}$.
Then $A_2$ is the union of $<\aleph_{\omega_1}$
many sets each of size $< \aleph_{\omega_1 + 1}$ $\lightning$.
Hence for some $\tilde{f}\colon T \to \aleph_\beta$,
\[
|A_3| = \aleph_{\omega_1 + 1},
\]
where $A_3 = \{Y \in A_2 : \overline{f}_Y\defon{T} = \tilde{f}\}$.
Let $Y, Y' \in A_3$ and $\alpha \in T$.
Then
\[
\overline{f}_Y(\alpha) = \overline{f}_{Y'}(\alpha),
\]
hence
\[
f_{\aleph_\alpha}(Y \cap \aleph_\alpha) = f_Y(\alpha) = f_{Y'}(\alpha) = f_{\aleph_\alpha}(Y' \cap \aleph_\alpha),
\]
i.e.~$Y \cap \aleph_\alpha = Y' \cap \aleph_\alpha$.
Since $T$ is cofinal in $\omega_1$,
it follows that $Y = Y'$.
So $|A_3| \le 1 \lightning$
\end{subproof}
Let us now define a sequence $\langle X_i : i < \aleph_{\omega_1 + 1} \rangle$
of subsets of $\aleph_{\omega_1 + 1}$ as follows:
Suppose $\langle X_j : j < i \rangle$
were already chosen.
Consider
\[
\{Y \subseteq \aleph_{\omega_1} : \exists j < i.~Y \le X_j\}
= \bigcup_{j < i} \{Y \subseteq \aleph_{\omega_1} : Y \le X_j\}.
\]
This set has cardinality $\le \aleph_{\omega_1}$
by \yaref{thm:silver:p:c3}.
Let $X_i \subseteq \aleph_{\omega_1}$
be such that $X_i \nleq X_j$ for all $j < i$.
The set
\[
\{Y \subseteq \aleph_{\omega_1} : \exists i < \aleph_{\omega_1 + 1} .~Y \le X_i\}
= \bigcup_{i < \aleph_{\omega_1 + 1}} \{Y \subseteq \aleph_{\omega_1} : Y \le X_i\}
\]
has size $\le \aleph_{\omega_1 + 1}$
(in fact the size is exactly $\aleph_{\omega_1 + 1}$).
But
\[
\{Y \subseteq \aleph_{\omega_1} : \exists i < \aleph_{\omega_1 + 1} .~Y \le X_i\} = \cP(\aleph_{\omega_1 + 1})
\]
because if $X \subseteq \aleph_{\omega_1 + 1}$
is such that $X \nleq X_i$ for all $i < \aleph_{\omega_1 + 1}$,
then $X_i \le X$ for all $i < \aleph_{\omega_1 + 1}$,
so such a set $X$ does not exist by \yaref{thm:silver:p:c3}.
\end{refproof}
2023-12-19 03:43:49 +01:00
>>>>>>> 5ee970194987e24c54bfc9c787cb219e15e71520